How to triangulate two 3d lines

Click For Summary
The discussion focuses on triangulating an object's 3D coordinates using two cameras in a computer vision project. Participants outline the equations of two lines representing the paths from each camera to the object, noting that these lines do not intersect. To find the object's location, the shortest line connecting the two lines must be calculated, with its midpoint representing the object's center. The transformation of coordinates between the original and camera-specific systems is also addressed, emphasizing the need for a change of basis matrix. The goal is to express the object's location accurately in 3D space relative to the cameras.
assafMOCAP
Messages
2
Reaction score
0
Hi guys,
I am working on an computer vision project.
the project uses two cameras to triangulate an object in front of the cameras.


Homework Statement


Express the object's location in 3d coordinates relative to the cameras.

Homework Equations


From the software i can get two line equations.
The line equations are for lines going through the centers of the lens and the center of the object itself
1: a1x1+b1y1+c1=0
2: a2x2+b2y2+c2=0
Known parameters are the a1,a2,b1,b2,c1,c2
Also know is the relation between the two cameras (The Fundamental matrix 3X3)
Also, the object coordinates in 2d on a projection plane from each camera is known (i.e. x1,y1 and x2,y2 )

Sadly, the lines do not meet, so its also needed to calculate the shortest line that connects both lines and treat its middle as the object's center.

The Attempt at a Solution


how to calculate a vector that originates from the middle of the two cameras, to the object.
 
Physics news on Phys.org
assafMOCAP said:

Homework Statement


Express the object's location in 3d coordinates relative to the cameras.

Let's say that your original coordinate system is ##(O,\vec i,\vec j,\vec k) ##
You want to set a camera in ##O'## and ##O''##, and set a coordinate system in ##O'## and ##O''##.

If ##M## has coordinates ##(x,y,z)## in the original coordinate system, its coordinates ##(x',y',z')## in ##(O',\vec I,\vec J,\vec K) ## satisfy

## \begin{pmatrix} x \\ y \\ z \end{pmatrix} = \vec{OO'} + P . \begin{pmatrix} x' \\ y' \\ z' \end{pmatrix} ##

where ##P## is the change of basis matrix from ##(\vec i,\vec j,\vec k)## to ##(\vec I,\vec J,\vec K) ##. Similarly for the other camera.
 
Question: A clock's minute hand has length 4 and its hour hand has length 3. What is the distance between the tips at the moment when it is increasing most rapidly?(Putnam Exam Question) Answer: Making assumption that both the hands moves at constant angular velocities, the answer is ## \sqrt{7} .## But don't you think this assumption is somewhat doubtful and wrong?

Similar threads

Replies
17
Views
2K
Replies
1
Views
1K
  • · Replies 1 ·
Replies
1
Views
2K
  • · Replies 1 ·
Replies
1
Views
5K
  • · Replies 11 ·
Replies
11
Views
3K
Replies
11
Views
5K
  • · Replies 1 ·
Replies
1
Views
7K
  • · Replies 1 ·
Replies
1
Views
3K
  • · Replies 2 ·
Replies
2
Views
2K
Replies
1
Views
2K